Jump to content

vermax

Members
  • Posts

    2
  • Joined

  • Last visited

Everything posted by vermax

  1. Thank you for your reply. I was familiar with the whole theory about Dirac delta function, but whole your text especially that example with sinus cleared things out for me perfectly now But if you would be kind enough I would need some more help in proving that equation: [math] \nabla \frac{ 1}{ | \vec{r} - \vec{r_0} |} = - 4 \pi \delta ( \vec{r} - \vec{r_0}) [/math] I have just figured out all that stuff from the link I have posted above and I think the method to achieve this: [math] \delta( \vec{r} - \vec{r_0})= \frac{1}{4 \pi r^2 }\delta ( r - r_0) [/math] is quite understandable now for me. But according to the excersie I have: [math] \nabla \frac{ 1}{ | \vec{r} - \vec{r_0} |} = - \frac{1}{r^2} \delta ( r - r_0) [/math] And I have just thought that if I counted this in spherical coordinates, gradient of my function 1/r should be also presented in that form. If thats ok I would have: [math]-\frac{1}{ | \vec{r} - \vec{r_0} |^2}= - \frac{1}{r^2} \delta ( r - r_0) [/math] And if I integrate both sides I get: [math]\frac{1}{ | \vec{r_0} - \vec{r} |}= \frac{1}{r_0^2} [/math] So rather it is not equal? :/ Uh I am confused with this... Merged post follows: Consecutive posts mergedUh I am sorry, dunno why but I write everywhere gradient, where there should be laplasian. As far as I see it will change my last equation so that on the left side we will have 1/(r0-r)^2... Sorry one more time.
  2. Hi guys. I play now a bit with EM fields and I have encountered some problems connected with Dirac delta. By coincidence I visited this forum and I thought I could find some help in here. Surely i have read these topics: http://www.scienceforums.net/forum/showthread.php?t=35186&highlight=dirac+delta http://www.scienceforums.net/forum/showthread.php?t=35132&highlight=dirac+delta That gave me a lot but i still do not feel it. The problem is that in order to get a potential in some point from a single charge you need to just solve such thing: [math]\square \phi = - \frac{ \rho}{\epsilon} [/math] and there by the way you need to use such equation: [math]\nabla \frac{ 1}{ | \vec{r} - \vec{r_0} |} = - 4 \pi \delta ( \vec{r} - \vec{r_0})[/math] I tried to find where it comes from, but the only thing I've found is this: http://www.fen.bilkent.edu.tr/~ercelebi/mp03.pdf But I do not get it at all I would appreciate if someone could just clear this out for me or present his own theory. PS. Sorry for my mistakes or improper names for some mathematical or physical stuff but I am from Poland
×
×
  • Create New...

Important Information

We have placed cookies on your device to help make this website better. You can adjust your cookie settings, otherwise we'll assume you're okay to continue.